This shows you the differences between two versions of the page.
Both sides previous revision Previous revision Next revision | Previous revision | ||
math104-f21:hw3 [2021/09/11 21:40] pzhou |
math104-f21:hw3 [2022/01/11 08:36] (current) pzhou ↷ Page moved from math104:hw3 to math104-f21:hw3 |
||
---|---|---|---|
Line 1: | Line 1: | ||
- | ====== HW 3 ====== | + | ====== HW 3 with solution |
- [Tao] Ex 5.4.3, | - [Tao] Ex 5.4.3, | ||
- [Tao] Ex 5.4.5 (you may assume result in 5.4.4) | - [Tao] Ex 5.4.5 (you may assume result in 5.4.4) | ||
Line 6: | Line 6: | ||
- Let be the subset of consisting of rational numbers with denominators of the form . Prove that for any , there is a Cauchy sequence in , such that . | - Let be the subset of consisting of rational numbers with denominators of the form . Prove that for any , there is a Cauchy sequence in , such that . | ||
+ | ====== Solution ====== | ||
+ | |||
+ | ==== 5.4.3 ==== | ||
+ | Problem: Show that for any real number , there is exactly one integer , such that . | ||
+ | |||
+ | Sol: If is an integer, then let . Assume is not an integer now. Then , hence by Prop 5.4.12, | ||
+ | |||
+ | |||
+ | Remark: we will call such the ' | ||
+ | |||
+ | ==== 5.4.5 ==== | ||
+ | Given any real numbers , show that there is a rational number such that . | ||
+ | |||
+ | Sol: Since , we can find a large enough positive integer , such that (by Archimedian property). Let be the unique integer, such that (proven in the problem 1), then , hence we have . Let $q = (m+1)/ | ||
+ | |||
+ | ==== 5.4.7 ==== | ||
+ | Let be real nubmers. Show that for all real $\epsilon> | ||
+ | |||
+ | Solution: If , then for any , we have , hence . In the other direction, if for all , we claim that . If not, say , then, we can let , then we get | ||
+ | c = (x-y) \leq \epsilon | ||
+ | which contradict with assumption , hence the claim is true. | ||
+ | |||
+ | For the second statement, if for all real , then by the same argument. Since we also have by definition of absolute value, hence . Thus . | ||
+ | |||
+ | ==== 5.4.8 ==== | ||
+ | Let be a Cauchy sequence of rationals, and let be a real number. Show that, if for all , then . Similarly, if for all , then . | ||
+ | |||
+ | Solution: We only prove the first half of the statement, since the other half is similar. We prove by contradiction, | ||
+ | |||
+ | |||
+ | ==== Problem 5 ==== | ||
+ | Let be the subset of consisting of rational numbers with denominators of the form . Prove that for any , there is a Cauchy sequence in , such that . | ||
+ | |||
+ | Solution: Let , and let be a Cauchy sequence in , such that , we are going to construct in , such that is equivalent to , hence . For , we let , | ||
+ | |||
+ | hence is equivalent to . | ||